Question

Given the following Cost Function: C(q) = 100 + 50q – 10q + q^3. Find the...

Given the following Cost Function: C(q) = 100 + 50q – 10q + q^3. Find the equations for Total Variable Costs, Average Variable costs, and Marginal Costs.

Suppose the Value of the firm is given by 14t1+rt , where the expected profit is 100, the interest rate is 3 percent. Derive the value of the firm.

0 0
Add a comment Improve this question Transcribed image text
Answer #1


Question 1

Total cost function is as follows -

TC = 100 + 50q - 10q2 + q3

The constant part of total cost function represents total fixed cost and the variable part represents total variable cost.

The constant part is 100.

The variable part is 50q - 10q2 + q3.

So,

The equation for total variable cost is as follows -

TVC = 50q - 10q2 + q3

Thus,

The equation for total variable cost is TVC = 50q - 10q2 + q3

Calculate the equation for average variable cost -

AVC = TVC/q

AVC = [50q - 10q2 + q3]/q

AVC = 50 - 10q + q2

Thus,

The equation for the average variable cost is AVC = 50 - 10q + q2.

Calculate the equation for marginal cost -

MC = dTC/dq

MC = [100 + 50q - 10q2 + q3]/dq

MC = 50 - 20q + 3q2

Thus,

The equation for marginal cost is MC = 50 - 20q + 3q2.

Add a comment
Know the answer?
Add Answer to:
Given the following Cost Function: C(q) = 100 + 50q – 10q + q^3. Find the...
Your Answer:

Post as a guest

Your Name:

What's your source?

Earn Coins

Coins can be redeemed for fabulous gifts.

Not the answer you're looking for? Ask your own homework help question. Our experts will answer your question WITHIN MINUTES for Free.
Similar Homework Help Questions
  • Consider a competitive firm with total costs given by TC(q) = 100 + 10q + q...

    Consider a competitive firm with total costs given by TC(q) = 100 + 10q + q 2 The firm faces a market price p = 50. (a) Write expressions for total revenue TR and marginal revenue MR as functions of output q. (b) Write expressions for average total cost ATC, average variable cost AVC, and marginal cost MC as functions of output q. (c) For what value of output is ATC minimized?

  • Consider a competitive rm with total costs given by TC(q) = 100 + 10q + q^2,...

    Consider a competitive rm with total costs given by TC(q) = 100 + 10q + q^2, The firm faces a market price p = 50. (a) Write expressions for total revenue TR and marginal revenue MR as functions of output q. (b) Write expressions for average total cost ATC, average variable cost AVC, and marginal cost MC as functions of output q. (c) For what value of output is ATC minimized? (d) Find the profit maximizing level of output q...

  • Suppose that the cost function of a firm is C(Q) = 490 + 10Q^2 . (a)...

    Suppose that the cost function of a firm is C(Q) = 490 + 10Q^2 . (a) Provide the mathematical expressions for AFC, AVC, AC and MC. (b) Graph all the costs above as a function of Q. (c) What is true about the relation between the marginal cost and the average costs when the latter are at their minimum? And when the average cost are increasing (decreasing)? Explain.

  • The cost to produce q electric cat brushes is described by the function: C(q)= 10q^2+250q, where...

    The cost to produce q electric cat brushes is described by the function: C(q)= 10q^2+250q, where q is hundreds of units for volumes less than 2,000. The demand function for electric cat bushes is described by: P(q)= -q^2-3q+1280, where p represents price in dollars. 1.) What are the company's marginal cost and marginal revenue functions? 2.) Calculate the number of units that produces the maximum profit. What price should the company charge and what is the maximum profit it will...

  • Suppose a firm has an average total cost function given by: ATC 9610/q +1 +10q Calculate...

    Suppose a firm has an average total cost function given by: ATC 9610/q +1 +10q Calculate the price where the firm breaks even (i.e. profit $0). (Do not include a $ sign in your response. Round to the nearest two decimal places if necessary.) Answer

  • 5. Consider the following cost function: c(q; F) = F + 10q + q2 , where2...

    5. Consider the following cost function: c(q; F) = F + 10q + q2 , where2 F > 0 represents the fixed cost: F = c(0; F). (a) Compute the marginal cost function, MC(q) = c0(q; F). (b) Show that the marginal cost function MC(q) is increasing. (c) Recall the average cost function, AC(q; F) = c(q;F) . Find qˉ(F),q the value of q (given F) at which AC(q; F) = MC(q).

  • 9. The Dijon Company's total variable cost function is TVC=50Q-10Q2 +Q3 where Q is the number...

    9. The Dijon Company's total variable cost function is TVC=50Q-10Q2 +Q3 where Q is the number of units of output produced. The total Fixed cost is $2000. a. What is the output level where marginal cost is a minimum? b. What is the output level where the average variable cost is a minimum? c. What is the value of average variable cost and marginal cost at the output specified in the answer to part (b)? d. What is the ATC...

  • Consider a competitive rm with total costs given by T C(q) = 100 + 10q +...

    Consider a competitive rm with total costs given by T C(q) = 100 + 10q + q 2 The rm faces a market price p = 50. (a) Write expressions for total revenue T R and marginal revenue MR as functions of output q. (b) Write expressions for average total cost AT C, average variable cost AV C, and marginal cost MC as functions of output q. (c) For what value of output is AT C minimized? 1 (d) Find...

  • need help with 5 and 6 Suppose a perfectly competitive firm's cost function is C(q)-4q*+16. Marginal cost for the firm...

    need help with 5 and 6 Suppose a perfectly competitive firm's cost function is C(q)-4q*+16. Marginal cost for the firm is given by MC=8q. 1) Find equations for variable cost, fixed cost, average total cost, average variable cost and average fixed cost for this firm. Illustrate on a graph the firm's average variable cost curve, average total cost curve, and marginal cost curve. 2) Find the outputs that minimize average total cost, average variable cost and average fixed cost. 3)...

  • Hi please write everything out legibly thank you Using the cost function TC 3 10Q+50, and...

    Hi please write everything out legibly thank you Using the cost function TC 3 10Q+50, and a production function Q KAL, K-85, L 120, determine the following 1. Quantity of output 2. Average product of labor 3. Average product of capital 4. Marginal product of labor 5 Marginal product of capital 6. Fixed cost and average fixed cost 7. Variable cost and Average variable cost 8. Total cost and average total cost 9 If capital rental rater 5 and wage...

ADVERTISEMENT
Free Homework Help App
Download From Google Play
Scan Your Homework
to Get Instant Free Answers
Need Online Homework Help?
Ask a Question
Get Answers For Free
Most questions answered within 3 hours.
ADVERTISEMENT
ADVERTISEMENT
ADVERTISEMENT